If V is more popular than Q and J is less popular than L, then which one of the following could be true of the ranking?

Sash on February 5, 2021

How is answer D even possible?

I'm not sure why the answer is J is more popular than V , when the question itself uses the condition that V is more popular than Q and J So obviously wouldn’t V come before J every time in order for it to be the more popular?

Reply
Create a free account to read and take part in forum discussions.

Already have an account? log in

Victoria on February 5, 2021

Hi @Sash,

Happy to help!

I think you just misread the question here. The question stem gives us two new conditions: (1) V is more popular than Q; and (2) J is less popular than L.

Therefore, it is entirely possible that J could be more popular than V as demonstrated by the following example.

H L J V Q S P
1 2 3 4 5 6 7

Notice that this meets all the original conditions and the conditions imposed by the question stem:

1) J (3) and L (2) are each less popular than H (1).
2) J (3) is more popular than Q (5).
3) S (6) and V (4) are each less popular than J (3).
4) P (7) and S (6) are each less popular than Q (5).
5) S is not seventh; S is sixth.
6) V (4) is more popular than Q (5).
7) J (3) is less popular than L (2).

Hope this helps! Please let us know if you have any further questions.